Solving Complex Exponential Equations: Finding Solutions to z^4 = -1

Click For Summary
The discussion focuses on solving the equation z^4 = -1 by expressing z in polar form as z = |z|e^{α*i}. Participants highlight that since z^4 = -1, it follows that z^2 = ±i, leading to the need to analyze the magnitude and angle of z. Key insights include the relationship between the magnitudes and angles of complex numbers, specifically that |z^2| = |i| = 1. The conversation emphasizes the importance of understanding the implications of negative values in complex exponentials to find all solutions. Ultimately, the original poster successfully resolves the problem after receiving hints and guidance.
mathman44
Messages
204
Reaction score
0

Homework Statement

Let z=|z|e^{\alpha*i}

Using the fact that z*w=|z||w|e^{i(\alpha+\beta)}, find all solutions to
z^4 = -1

The Attempt at a Solution



Not quite sure how to proceed, except for the obvious step

i=z^2=|z*z|e^{i(2\alpha)}= |z*z|[cos(2\alpha)+isin(2\alpha)]

Kinda stuck here :s any hints? Thanks.
 
Physics news on Phys.org
Just some hints you might have overlooked.

|z*z| = |(-z)*(-z)|

i^2=(z^2)^2=(-(z^2))^2
 
transphenomen said:
Just some hints you might have overlooked.

|z*z| = |(-z)*(-z)|

i^2=(z^2)^2=(-(z^2))^2

Ok so I can replace

|z*z|[cos(2\alpha)+isin(2\alpha)]

with [cos(2\alpha)+isin(2\alpha)]

since |z*z|=|z^2|=|i|=1
 
mathman44 said:
Ok so I can replace

|z*z|[cos(2\alpha)+isin(2\alpha)]

with [cos(2\alpha)+isin(2\alpha)]

since |z*z|=|z^2|=|i|=1

Yup, but it is those negatives that allow you to get more z's that solve the problem z^4=i
 
mathman44 said:

Homework Statement




Let z=|z|e^{\alpha*i}

Using the fact that z*w=|z||w|e^{i(\alpha+\beta)}, find all solutions to
z^4 = -1

The Attempt at a Solution



Not quite sure how to proceed, except for the obvious step

i=z^2=|z*z|e^{i(2\alpha)}= |z*z|[\cos(2\alpha)+\isin(2\alpha)]

Kinda stuck here :s any hints? Thanks.
For one thing, if z^4 = -1\,, then z^2 = \pm i\,.

Seems that the hint might be more helpful had it said:

\text{If }\, z=\left|z\right|e^{\alpha\cdot i}\text{ and }w=\left|w\right|e^{\beta\cdot i}, \text{ then } z\cdot w=\left|z\right|\left|w\right| e^{(\alpha+\beta)i}\,.

\text{Also, }\ -1=\left|-1\right|e^{\pi i}=1e^{\pi i}=e^{\pi i}\,.

\text{and, }\ i=\left|i\right|e^{\pi i/2}=e^{\pi i/2}\,.

Added in edit: Additional helpful facts.

e^{2\pi n\,i}=\left(e^{2\pi\,i}\right)^n=\left(1\right)^n=1\,,\ \text{ where n is an integer.}

-1=e^{\pi i}\cdot e^{2\pi n\,i}=e^{(2\pi n+\pi)\,i}

\left|w\right|e^{i\beta}=\left|v\right|e^{i\phi}\ \ \implies\ \ \left\{\left|w\right|=\left|\ v\right| \ \text{ and }\ \beta =\phi\ \right\}.

Solve: \left(\left|z\right|e^{i\alpha}\right)^4=e^{(2\pi n+\pi)\,i}\,.
 
Last edited:
Thanks very much Sammy, I've figured it out :)
 
Question: A clock's minute hand has length 4 and its hour hand has length 3. What is the distance between the tips at the moment when it is increasing most rapidly?(Putnam Exam Question) Answer: Making assumption that both the hands moves at constant angular velocities, the answer is ## \sqrt{7} .## But don't you think this assumption is somewhat doubtful and wrong?

Similar threads

Replies
3
Views
2K
  • · Replies 2 ·
Replies
2
Views
3K
Replies
7
Views
2K
Replies
13
Views
2K
  • · Replies 2 ·
Replies
2
Views
1K
  • · Replies 6 ·
Replies
6
Views
2K
  • · Replies 3 ·
Replies
3
Views
2K
  • · Replies 17 ·
Replies
17
Views
2K
  • · Replies 8 ·
Replies
8
Views
2K